Gravitations-Rotverschiebung um ein Schwarzschild-Schwarzes Loch

Nehmen wir an, ich schwebe in einer Rakete bei konstanten räumlichen Koordinaten außerhalb eines Schwarzschild-Schwarzen Lochs.

Ich lasse eine Glühbirne in das Schwarze Loch fallen, und sie strahlt etwas Licht in einer Entfernung von R e aus der Mitte, mit einer Wellenlänge von λ e im Ruherahmen der Glühbirne.

Welche Wellenlänge hätte das Licht, wenn es mich erreicht? R Ö B S in Bezug auf den Radius, in dem es emittiert wird, R e ?

Dies ist eine Unterfrage aus Sean Carrolls Spacetime and Geomtery . Zu Beginn des Kapitels behauptet Carroll, dass jeder stationäre Beobachter ( U ich = 0 ) misst die Frequenz eines Photons nach einer Null-Geodäte X μ ( λ ) sein

ω = G μ v U μ D X v D λ

Ich verstehe nicht, woher dieser Ausdruck kommt. Wie kann man Dinge wie Wellenlänge und Frequenz des Lichts überhaupt in Bezug auf allgemeine relativistische Größen wie konzeptualisieren? U , G μ v , D S 2 , usw?

Ins Schwarze Loch? Oder meinen Sie näher am Schwarzen Loch, aber immer noch außerhalb des Ereignishorizonts?
Nicht direkt in die Singularität gefallen, aber nicht unbedingt außerhalb des Ereignishorizonts. Es kann der Fall sein, dass die Glühbirne außerhalb des Ereignishorizonts fallen gelassen wird und dann Licht von innen emittiert, oder dass sie innerhalb des Ereignishorizonts fallen gelassen wird und dort ebenfalls Licht emittiert, oder dass sie von außerhalb des Ereignishorizonts fallen gelassen wird und strahlt Licht nach außen. Außerdem nehmen wir an, dass der Pfad, dem die Birne folgt, rein radial ist und dass die Beobachterkoordinaten stationär sind.
Ja, „ins Schwarze Loch“ bedeutet „innerhalb des Ereignishorizonts“. Sie wissen, dass Licht innerhalb des Ereignishorizonts nicht entkommen kann, richtig?
Ja – also sollte es eine mathematische Beziehung zwischen dem Radius, bei dem das Licht emittiert wird, und der Wellenlänge des beobachteten Lichts geben, die unendlich oder undefiniert für Emissionspunkte von weniger als 2 GM und wohldefiniert für Emissionspunkte von mehr als 2 GM ist – I ' Ich versuche nur zu verstehen, wie diese Beziehung abgeleitet werden würde.
Und Entschuldigung! Der Beobachter sollte sich außerhalb des Schwarzen Lochs befinden – es gibt jedoch noch keine Beschränkungen für den Emissionspunkt des Lichts.
OK ... es macht einfach keinen Sinn, speziell nach dem Fall zu fragen, in dem Licht von innerhalb des Ereignishorizonts emittiert wird und einen Beobachter außerhalb erreicht, da dies nicht der Fall ist. Ich denke, Sie werden viel wahrscheinlicher die gewünschten Antworten erhalten, wenn Sie die Bedingung fallen lassen, dass sich die Glühbirne im Inneren des Schwarzen Lochs befindet.
Gravitations-Rotverschiebung ist der Begriff, nach dem man googeln muss! Die Ableitung der Formel erfolgt in allen guten GR-Lehrbüchern.
@DavidZaslavsky Es ist nicht erforderlich, die Anfangsbedingungen auf außerhalb des Ereignishorizonts zu beschränken. Die Metrik liefert die richtige Lösung.
@zhermes Ja, ich weiß, ich sage nur, dass es nicht nötig ist, sie auf den Ereignishorizont zu beschränken .

Antworten (3)

Hier sind einige Ideen zu Ihrer Frage:

Betrachten wir den Weg, den die Fackel in Gegenwart eines Schwarzen Lochs nimmt, und nehmen wir an, der Beobachter befindet sich außerhalb des Horizonts. Nehmen wir der Einfachheit halber an, dass die Lichtquelle (die Taschenlampe) geradlinig fällt.

Etwas Algebra und ziemlich viel Physik , die das Äquivalenzprinzip und einige Aspekte der speziellen Relativitätstheorie kombinieren, können zeigen, dass die Geometrie des Fackelpfads durch die Gleichung gegeben ist (wobei nur geradlinige Bewegung berücksichtigt wird):

D S 2 = ( 1 2 G M / C 2 R ) C 2 D T 2 D R 2 ( 1 2 G M / C 2 R ) .

Wobei r der Abstand der Fackel vom Mittelpunkt des BH und M die Masse des BH ist. Die Koeffizienten von D T 2 Und D R 2 sind die metrischen „Tensorkomponenten“ der Raum-Zeit-Geometrie für diese spezielle Fragestellung. Für das Licht der Fackel ist der Weg jedoch eine geodätische Kurve – Linie des kürzesten Weges, den das Licht in einer enorm gekrümmten Raumzeit nimmt, und die obige Gleichung wird D S 2 = 0 somit:

( 1 2 G M / C 2 R ) C 2 D T 2 D R 2 ( 1 2 G M / C 2 R ) = 0 .

Die letztere Gleichung gibt die Geschwindigkeit der Lichtquelle, der Fackel, an, wenn sie von außerhalb des Horizonts auf den BH fällt und vom Beobachter in einer sehr großen Entfernung vom BH beobachtet wird

v ( R ) = C ( 1 2 G M / C 2 R ) .

Die Frequenzverschiebung z = F F 0 F 0 bezieht sich über die Gleichung auf die Geschwindigkeit der Lichtquelle

v ( R ) = C z 2 + 2 z z 2 + 2 z + 2 .

Die letzte Gleichung gibt an, wie sich die Frequenzverschiebung in Abhängigkeit von ändert R , und wie sie von der Masse M des BH beeinflusst wird. Hier, F ist die vom Beobachter empfangene Frequenz, während F 0 ist die tatsächliche Frequenz, die von der Lichtquelle (der Taschenlampe) ausgestrahlt wird.

Danke dafür, Johannes! Aber in diesem Szenario gehen wir tatsächlich davon aus, dass sich der Beobachter nicht sehr weit vom Schwarzen Loch entfernt befindet, sondern an konstanten, sich mitbewegenden räumlichen Koordinaten an einer Rakete aufgehängt ist ( R Ö B S ) außerhalb des Ereignishorizonts. R Ö B S ist nicht weit genug vom BH entfernt, um als ebener Platz genommen zu werden.
Nur zum Spitzhacken, R ist nicht "die Entfernung der Fackel vom Zentrum des BH", sondern nur eine Koordinate. Es entspricht nicht radialen Abständen.
@danig Wenn ich große Entfernung vom BH sage, meine ich nicht wirklich unendlich. Ich meine, der Beobachter fällt nicht mit der Fackel...
@elfmotat Wie ist das? Die Herleitung dieser "einfachen Gleichung" basiert auf der Annahme, dass R ist der radiale Abstand vom Gravitationskörper, der sich zufällig in ein BH verwandelt hat. Sie müssen definieren, was Sie mit "nur eine Koordinate" meinen, Koordinaten beziehen sich normalerweise auf einen Referenzpunkt, sonst machen sie keinen Sinn.
Sie verwenden Schwarzschild-Koordinaten,
Sie verwenden Schwarzschild-Koordinaten. In diesen Koordinaten R entspricht nicht radialen Abständen. Die physikalische Entfernung, die Sie mit einem Lineal messen würden, entspricht einem Integral über D S . 99 % aller Verwirrung in GR kommt davon, dass Koordinaten zu ernst genommen werden.
@elfmotat Ja, man muss einige Koordinaten verwenden, und Schwarzschild-Koordinaten sind gut genug für ein statisches und kugelsymmetrisches BH. Schließlich lassen sich auch diese in Kruskal-Szekeres-Koordinaten umwandeln. Ich verstehe jedoch, worauf Sie hinauswollen, und dafür danke ich Ihnen.

Es wäre hilfreich gewesen, wenn das OP einen Hinweis darauf hätte geben können, wo dieses Material in Carroll vorkommt. Ich habe die kostenlose arxiv-Version durchgesehen (es gibt auch eine kostenlose HTML-Version ) und konnte sie nicht finden. Die Frage ist auch ein wenig weit gefasst und vage, was der Hintergrund des OP ist. Die Frage besteht im Wesentlichen aus zwei Teilen: (1) Begründung und Interpretation der bei Carroll angegebenen Form für den Rotverschiebungsfaktor und (2) Anwendung dieser auf das Problem der fallenden Glühbirne.

Zu Nr. 2, ich denke, das Folgende funktioniert als heuristische Ableitung der Form der von Carroll angegebenen Beziehung und kann helfen, ihren physikalischen Inhalt und die Bedeutung der Mathematik zu erklären. Der Energie-Impuls-Viervektor eines Teilchens P ist proportional zu seinem Geschwindigkeitsvierervektor v . Das liegt im Grunde daran, dass es eine andere Richtung geben könnte, in die es zeigen könnte? (Es macht auch Sinn, um der Newtonschen Beziehung korrekt zu entsprechen P = M v .) Nun, ein Lichtstrahl hat eigentlich keinen normalisierbaren Geschwindigkeits-Vier-Vektor, aber das ist in Ordnung, weil wir nur mit Proportionalitäten arbeiten. Wir brauchen keine Normalisierung. Wir wissen auch, dass die Frequenz vier Vektor ist ω ist proportional zu P . Das hat rein klassische Gründe, ist aber für den modernen Menschen einfacher zu begründen, weil die Proportionalitätskonstante bei einem einzelnen Photon die Plancksche Konstante ist.

In Indexschreibweise angegeben, haben wir ω A v A . Dies bezieht sich auf die kontravariante Form der Frequenz, aber normalerweise arbeiten wir mit ihrer Kovektorform, die so definiert ist, dass für einen Beobachter mit Geschwindigkeit U , die Rate ω (Skalar), an dem Wellenfronten ankommen, ist ω = U A ω A . Wir haben daher ω U A v A , was im Wesentlichen Carroll zu sagen scheint (obwohl der Kontext fehlt).

Der sign in Carroll ist nur eine Proportionalitätskonstante, also können wir sie ignorieren. Vermutlich hat er es da, weil er in der arbeitet + + + metrisch und er möchte, dass dies positiv herauskommt. Der G μ v ist da, weil er einen Index senkt. Die Tatsache, dass der Beobachter stationär ist, wirkt sich in keiner Weise auf das Argument der Proportionalität aus, aber vermutlich ist diese Annahme für den Fall, den er betrachtet (möglicherweise eine Dopplerverschiebung, die sich auf einen stationären Beobachter im Unendlichen bezieht), notwendig, um die Proportionalität zu machen beständig sei derjenige, den er gibt. Die Wahl des affinen Parameters λ ist willkürlich, und wieder ist es ein wenig schwer zu wissen, wie Carroll beabsichtigt, diese Mehrdeutigkeit aufzulösen, wenn er die Konstante der Proportionalität angibt, da jede Neudefinition des affinen Parameters λ A λ + B ändert das Ergebnis um den Faktor A . (Und da dies eine Null-Geodäte ist, gibt es keine natürliche Wahl für den affinen Parameter, wie z. B. eine Eigenzeit.)

Zu Nr. 1, ich denke, wir müssten die tatsächliche Beschreibung des Problems sehen, da einige Dinge ausgelassen zu werden scheinen. Insbesondere ist mir nicht klar, ob die Taschenlampe aus der Ruhe bei Unendlich oder aus der Ruhe bei gefallen ist R obs . In beiden Fällen müssen Sie wahrscheinlich den Geschwindigkeits-Vier-Vektor finden, wenn er erreicht wird R e , aber das wäre eine separate Berechnung. Auch wird nicht angegeben, ob der Beobachter ruht oder sich relativ zum Schwarzen Loch bewegt. Im Ruhezustand muss sich der Beobachter außerhalb des Horizonts befinden, und der Blitz kann nicht beobachtet werden, wenn er von innerhalb des Horizonts emittiert wurde.

Lassen Sie mich auf Ihre Teilfrage eingehen. An früherer Stelle im Text sagt Carroll, dass ein Beobachter mit Geschwindigkeit U μ misst die Energie eines Teilchens entlang einer Geodäte E = P μ U μ . Wir können diese Beziehung ableiten, indem wir zu lokal flachen Koordinaten mit Metrik wechseln η μ v und unter der Annahme eines stationären Beobachters U μ = ( 1 , 0 , 0 , 0 ) . Hier, P μ U μ = η μ v P μ U v = η 0 0 P 0 U 0 . Verwenden E = P 0 Und η 00 = 1 liefert uns das gewünschte Ergebnis. Da es sich um eine Tensorgleichung handelt, muss sie in allen Koordinatensystemen gelten. Jetzt verwenden wir P v = D X v ( λ ) D λ , E = ω = G μ v U μ D X v ( λ ) D λ und einstecken = 1 um das gewünschte Ergebnis zu erhalten.